Sei sulla pagina 1di 21

LAND TITLES AND DEEDS CASES

NON-REGISTRABLE PROPERTIES A. THOSE DEVOTED FOR PUBLIC SERVICES 1. USE AND

ISSUES: (1) Can the Roppongi property and others of its kind be alienated by the Philippine Government?; and (2) Does the Chief Executive, her officers and agents, have the authority and jurisdiction, to sell the Roppongi property? HELD: Petitions were granted.

SALVADOR LAUREL vs. RAMON GARCIA, head of the Asset Privatization Trust, DFA Sec. RAUL MANGLAPUS, and E.S. CATALINO MACARAIG (G.R. No. 92013) 25 July 1990; and DIONISIO OJEDA vs. E.S. MACARAIG, APT CHAIR RAMON GARCIA, AMB. RAMON DEL ROSARIO, et al., as members of the PRINCIPAL AND BIDDING COMMITTEES ON THE UTILIZATION/DISPOSITION PETITION OF PHIL/ GOVT PROPERTIES IN JAPAN, (G.R. No. 92047) 25 July 1990

The nature of the Roppongi lot as property for public service is expressly spelled out. It is dictated by the terms of the Reparations Agreement and the corresponding contract of procurement which bind both the Philippine government and the Japanese government. There can be no doubt that it is of public dominion unless it is convincingly shown that the property has become patrimonial. This, the respondents have failed to do. As property of public dominion, the Roppongi lot is outside the commerce of man. It cannot be alienated. Its ownership is a special collective ownership for general use and enjoyment, an application to the satisfaction of collective needs, and resides in the social group. The purpose is not to serve the State as a juridical person, but the citizens; it is intended for the common and public welfare and cannot be the object of appropration. The Roppongi property is correctly classified under paragraph 2 of Article 420 of the Civil Code as property belonging to the State and intended for some public service. The fact that the Roppongi site has not been used for a long time for actual Embassy service does not automatically convert it to patrimonial property. The issues are not concerned with validity of ownership or title. There is no question that the property belongs to the Philippines. The issue is the authority of the respondent officials to validly dispose of property belonging to the State. And the validity of the procedures adopted to effect its sale. This is governed by Philippine Law. The rule of lex situs does not apply. It is not for the President to convey valuable real property of the government on his or her own sole will. Any such conveyance must be authorized and approved by a law enacted by the Congress. It requires executive and legislative concurrence. B. MINERAL LANDS 2. Republic v. CA and dela Rosa (GR L-43938, 15 April 1980, 160 SCRA 228)

FACTS: These are two petitions for prohibition seeking to enjoin respondents, their representatives and agents from proceeding with the bidding for the sale of the 3,179 square meters of land at 306 Roppongi, 5-Chome Minato-ku Tokyo, Japan scheduled on February 21, 1990. The subject property in this case is one of the four (4) properties in Japan acquired by the Philippine government under the Reparations Agreement entered into with Japan on May 9, 1956. August 11, 1986, President Aquino created a committee to study the disposition/utilization of Philippine government properties in Tokyo and Kobe, Japan through Administrative Order No. 3, followed by Administrative Orders Numbered 3-A, B, C and D, despite growing unfavorable sentiment among the people. On July 25, 1987, the President issued Executive Order No. 296 entitling non-Filipino citizens or entities to avail of separations' capital goods and services in the event of sale, lease or disposition. The petitioner in G.R. No. 92013 objects to the alienation of the Roppongi property to anyone while the petitioner in G.R. No. 92047 adds as a principal objection the alleged unjustified bias of the Philippine government in favor of selling the property to non-Filipino citizens and entities. These petitions have been consolidated and are resolved at the same time for the objective is the same to stop the sale of the Roppongi property. Laurel states that the Roppongi property is classified as one of public dominion, and not of private ownership under Article 420 of the Civil Code. Noting the non-use of the Roppongi property at the moment, the petitioner avers that the same remains property of public dominion so long as the government has not used it for other purposes nor adopted any measure constituting a removal of its original purpose or use. The respondents, for their part, refute the petitioner's contention by saying that the subject property is not governed by our Civil Code but by the laws of Japan where the property is located. They rely upon the rule of lex situs which is used in determining the applicable law regarding the acquisition, transfer and devolution of the title to a property. They also add that the propertiy has become patrimonial property because it has not been used for public service or for diplomatic purposes for over thirteen (13) years now.

FACTS: Several parties were claiming ownership of the land, situated in Tuding, Itogon, Benguet Province. Jose de la Rosa, in his capacity as private individual, claimed said land as agricultural land which was acquired by them in 1964. Two companies were also claiming the land as mining land. They alleged that the location of the mining claim under consideration was perfected prior to November 15, 1935.

emm / LTD cases, page 1

The Bureau of Forestry Development also argued that the land sought to be registered was covered by the Central Cordillera Forest Reserve under Proclamation No. 217 dated February 16, 1929. Moreover, by reason of its nature, it was not subject to alienation under the Constitutions of 1935 and 1973. The trial court denied the application, holding that the applicants had failed to prove their claim of possession and ownership of the land sought to be registered. The applicants appealed to the respondent court, which reversed the trial court and recognized the claims of the applicant, but subject to the rights of Benguet and Atok respecting their mining claims. In other words, the Court of Appeals affirmed the surface rights of the de la Rosas over the land while at the same time reserving the subsurface rights of Benguet and Atok by virtue of their mining claims. ISSUES: WoN a surface rights over a certain land can be treated separately from its sub-surface rights. RULING: The June Bug mineral claim of Benguet and the Fredia and Emma mineral claims of Atok having been perfected prior to the approval of the Constitution of the Philippines of 1935, they were removed from the public domain and had become private properties of Benguet and Atok. "The legal effect of a valid location of a mining claim is not only to segregate the area from the public domain, but to grant to the locator the beneficial ownership of the claim and the right to a patent therefor upon compliance with the terms and conditions prescribed by law. Where there is a valid location of a mining claim, the area becomes segregated from the public domain and the property of the locator." It is of no importance whether Benguet and Atok had secured a patent for as held in the Gold Creek Mining Corp. Case, for all physical purposes of ownership, the owner is not required to secure a patent as long as he complies with the provisions of the mining laws; his possessory right, for all practical purposes of ownership, is as good as though secured by patent. We agree likewise with the oppositors that having complied with all the requirements of the mining laws, the claims were removed from the public domain, and not even the government of the Philippines can take away this right from them. The reason is obvious. Having become the private properties of the oppositors, they cannot be deprived thereof without due process of law. Act No. 4268, approved on November 8, 1935, declared: Any provision of existing laws, executive order, proclamation to the contrary notwithstanding, all locations of mining claim made prior to February 8, 1935 within lands set apart as forest reserve under Sec. 1826 of the Revised Administrative Code which would be valid and subsisting location except to the existence of said reserve are hereby declared to be valid and subsisting locations as of the date of their respective locations. However, the method invoked by the de la Rosas is not available in the case at bar, for two reasons. First, the trial court found that the evidence is insufficient and that they had acquired the land only in 1964 and

applied for its registration in 1965, relying on the earlier alleged possession of their predecessors-in-interest. Second, The property was mineral land, and they claiming it as agricultural land. Nevertheless, they not disputing the lights of the mining locators nor they seeking to oust them as such and to replace in the mining of the land. were were were them

The Court feels that the rights over the land are indivisible and that the land itself cannot be half agricultural and half mineral. The classification must be categorical; the land must be either completely mineral or completely agricultural. In the instant case, as already observed, the land which was originally classified as forest land ceased to be so and became mineral and completely mineral once the mining claims were perfected. As long as mining operations were being undertaken thereon, or underneath, it did not cease to be so and become agricultural, even if only partly so, because it was enclosed with a fence and was cultivated by those who were unlawfully occupying the surface. An application of the Regalian doctrine which, as its name implies, is intended for the benefit of the State, not of private persons. The rule simply reserves to the State all minerals that may be found in public and even private land devoted to "agricultural, industrial, commercial, residential or (for) any purpose other than mining." Thus, if a person is the owner of agricultural land in which minerals are discovered, his ownership of such land does not give him the right to extract or utilize the said minerals without the permission of the State to which such minerals belong. The correct interpretation is that once minerals are discovered in the land, whatever the use to which it is being devoted at the time, such use may be discontinued by the State to enable it to extract the minerals therein in the exercise of its sovereign prerogative. The land is thus converted to mineral land and may not be used by any private party, including the registered owner thereof, for any other purpose that will impede the mining operations to be undertaken therein, For the loss sustained by such owner, he is of course entitled to just compensation under the Mining Laws or in appropriate expropriation proceedings. C. FOREST LANDS 3. Republic v. CA and Lastimado (GR 39473, 30 April 1979, 89 SCRA 648)

This is a Petition for Review (Appeal) by certiorari filed by RP from the CAs Decision on 30 Sept. 1974 in denying the State's Petition for certiorari and Mandamus. Private respondent, Isabel Lastimado, filed on 11 Sept. 1967, in the CFI-Bataan, a Petition for the reopening of cadastral proceedings over a portion of Lot No. 626 of the Mariveles Cadastre, consisting of 971 has., pursuant to RA 931, as amended by RA 2061. In the absence of any opposition, the Trial Court granted her Petition on 14 Oct. 1967. The Trial Court issued an order for the issuance of a decree of registration on 20 Nov. 1967, and on 21 Nov. 1967, the Land Registration Commission issued Decree No. N-117573 in favor of Lastimoso. Eventually, OCT No. N-144 was also issued in her favor. She thereafter subdivided the land into 10 lots, with corresponding titles. TCT Nos. 18905 to 18914 inclusive, were issued by the Register of Deeds.
emm / LTD cases, page 2

On 3 June 1968, or within 1 year from the entry of the decree of registration, petitioner filed a Petition for Review pursuant to Sec. 38, Act 496, on the ground of fraud alleging that during the period of alleged adverse possession by private respondent, said parcel of land was part of the U.S. Military Reservation in Bataan. which was formally turned over to RP only on 22 Dec. 1965, and that the same is inside the public forest of Mariveles, Bataan and, therefore, not subject to disposition or acquisition under the Public Land Law. Respondent field an Opposition thereto, which was considered by the Trial Court, as a Motion to Dismiss, and on 20 Dec. 1968, said Court issued an Order dismissing the Petition for Review mainly on the ground that the SolGen had failed to file opposition to the original Petition for reopening of the cadastral proceedings and was, therefore, estopped from questioning the decree of registration ordered issued therein. On 28 Jan. 1969, petitioner moved for reconsideration, which was denied by the Trial Court in its Order dated 20 May 1969, for lack of merit. Petitioner seasonably filed a Notice of Appeal and a Record on Appeal, which was objected to by private respondent. On 15 July 1972, or 3 years later, the trial Court (Judge Abraham P. Vera) refused to give due course to the appeal. Petitioner filed an MR but the Trial Court denied it in its Order of 14 Oct. 1972 on the ground that the proper remedy of petitioner was a certiorari petition, not an ordinary appeal, and that the Order sought to be appealed from had long become final and executory as petitioner's MR was pro-forma and did not suspend the running of the reglementary period of appeal. On 9 Nov. 1972, petitioner filed a Petition for certiorari and mandamus with the CA claiming that the Trial Court gravely abused its discretion, amounting to lack of jurisdiction when, without the benefit of hearing, it summarily dismissed the Petition for Review; and since said Petition raised certain issues of fact which cannot be decided except in a trial on the merits, the dismissal of the Petition on the basis of private respondent's Opposition, considered as a Motion to Dismiss, constituted a denial of due process of law. Petitioner then prayed that the Order of the trial Court, dated 20 Dec. 1968 dismissing the Petition for Review, be declared null and void, and that said Trial Court be directed to give due course to the Petition for Review; or, in the alternative, to give due course to petitioner's appeal. On 30 Sept. 1974, the CA upheld the Trial Court's dismissal of the Petition for Review stating that it cannot find any allegation in the petition for review which shows that private respondent had committed fraud against petitioner. From this Decision, petitioner filed the present Petition for Review (Appeal) by certiorari assigning the following errors to the CA and to the Lower Court (LC): 1. LC and CA erred in finding that there can be possession, even for the purpose of claiming title, of land which at the time of possession is subject to a military reservation. 2. LC and CA erred in finding that such land which is subject to a government reservation, may appropriately be the subject of cadastral proceedings, and hence, also of a petition to reopen cadastral proceedings.

3. LC and CA erred in finding that a parcel of land which is part of the public forest is susceptible of occupation and registration in favor of private individual. 4. LC and CA erred in not finding that the RP is not estopped from questioning the decree of registration and the title issued pursuant thereto in favor of respondent Lastimado over the parcel of land in question. 5. LC erred in dismissing the petition for review of the RP. 6. CA erred in denying Petitioner's petition for certiorari and mandamus. The essential elements for the allowance of the reopening or review of a decree are: a) that the petitioner has a real and dominical right; b) that he has been deprived thereof; c) through fraud; d) that the petition is filed within one year from the issuance of the decree; and e) that the property has not as yet been transferred to an innocent purchaser. However, for fraud to justify the review of a decree, it must be extrinsic or collateral and the facts upon which it is based have not been controverted or resolved in the case where the judgment sought to be annulled was rendered. The following ruling spells out the difference between extrinsic and intrinsic fraud: Extrinsic or collateral fraud, as distinguished from intrinsic fraud, connotes any fraudulent scheme executed by a prevailing litigant "outside the trial of a case against the defeated party, or his agents, attorneys or witnesses, whereby said defeated party is prevented from presenting fully and fairly his side of the case." But intrinsic fraud takes the form of "acts of a party in a litigation during the trial such as the use of forged instruments or perjured testimony, which did not affect the present action of the case, but did prevent a fair and just determination of the case. The fraud is one that affects and goes into the jurisdiction of the Court. In its Petition for Review filed before the trial Court, petitioner alleged that Ms. L committed fraud when she misrepresented that she and her predecessors-ininterest had been in possession of the land publicly, peacefully, exclusively and adversely against the whole world as owner for more than 40 years when, in fact, the subject land was inside the former U.S. Military Reservation, which was formally turned over to RP only on 22 Dec. 1965, and that she likewise contended that her rights, as derived from the original and primitive occupants of the land in question, are capable of judicial confirmation under existing laws, when the truth is, said parcel of land is within the public forest of Mariveles, Bataan, and is not subject to disposition or acquisition by private persons under the Public Land Law. The Trial Court ruled, and was upheld by the CA, that no fraud was committed by Ms. L, which deprived petitioner of its day in Court as there was no showing that she was aware of the facts alleged by the Government, so that she could not have suppressed them with intent to deceive. The Trial Court also noted that petitioner had failed to file an opposition to the reopening of the cadastral proceedings despite notices sent not only to the SolGen as required by RA 931, but to the Bureau of Lands and the Bureau of Forestry as well. It then concluded that "the remedy granted by Sec. 38 of the
emm / LTD cases, page 3

Land Registration Act is designed to give relief to victims of fraud, not to those who are victims of their own neglect, inaction or carelessness, especially when no attempt is ever made to excuse or justify the neglect." With the foregoing as the essential basis, the trial Court dismissed the Petition for Review. SC finds reversible error. Although there was an agreement by the parties to submit for resolution the Opposition to the Petition for Review, which was treated as a motion to dismiss, the Trial Court should not have dismissed the Petition outright but should have afforded petitioner an opportunity to present evidence in support of the facts alleged to constitute actual and extrinsic fraud committed by private respondent. Thus, in the case of Republic vs. Sioson, et al., it was held that "the action of the lower Court in denying the petition for review of a decree of registration filed within one year from entry of the decision without hearing the evidence in support of the allegation and claim that actual and extrinsic fraud upon which the petition is predicated, is held to be in error, because the lower Court should have afforded the petitioner an opportunity to prove it." If the allegation of petitioner that the land in question was inside the military reservation at the time it was claimed is true, then, it cannot be the object of any cadastral partition nor can it be the object of reopening under RA 931. Similarly, if the land in question, indeed forms part of the public forest, then, possession thereof, however long, cannot convert it into private property as it is within the exclusive jurisdiction of the Bureau of Forestry and beyond the power and jurisdiction of the Cadastral Court to register under the Torrens System. Even assuming that the government agencies can be faulted for inaction and neglect (although the SolGen claims that it received no notice), yet, the same cannot operate to bar action by the State as it cannot be estopped by the mistake or error of its officials or agents. Further, we cannot lose sight of the cardinal consideration that "the State as persona in law is the juridical entity, which is the source of any asserted right to ownership in land" under basic Constitutional Precepts, and that it is moreover charged with the conservation of such patrimony. The CAs Decision dated 30 Sept. 1974, dism issing the Petition for certiorari and mandamus filed before it, as well as the Order of the CFI-Bataan dated 20 Dec. 1968, dismissing the Petition for Review, are hereby set aside and the records of this case hereby remanded to the latter Court for further proceedings to enable petitioner to present evidence in support of its Petition for Review. No pronouncement as to costs. 4. Director of Lands v. Abanzado (GR L-21814, 15 July 1975, 65 SCRA 5)

the administration of the Director of Forestry and that said lot also forms a portion of Tanjay Cadastre, which was declared public land. Allegedly, sometime in 1959, the respondents-spouses, with the intention of defrauding the government, filed an answer in Cad. Case No. N-4 claiming Lot No. 6034 as their property through long and adverse possession and that on December 29, 1959, the Cadastral Court rendered judgment adjudicating said Lot No. 6034 to respondentsspouses, but no decree of registration has as yet been entered by the Land Registration Commission pursuant to said judgment. The Director of Forestry argued that they were not duly notified of the hearing over said lot and so they were unable to oppose its registration in the name of respondents-spouses. Director of Forestry also claimed that Lot 6034 is nondisposable land, the same being a part of the Baia Communal Forest, no portion of which has been released as disposable by the Bureau of Forestry in favor of the Bureau of Lands or any other person or entity. Respondents filed their opposition, they were sustained. The lower court disregarded the contention that at the very least, the Director of Forestry was entitled to be heard on his petition and was equally deaf to the plea that non-disposable timberland was involved. Thus this petition. ISSUE: Whether the disputed lot is non- registrable. HELD: Relying on due process, that instead of an outright denial of the petition for review, the lower court should have set the matter for hearing to enable the Bureau of Forestry to prove its claim. While the lower court was not legally bound to send personal notice of the hearing to appellant, it does not mean, however, that it acted correctly and within legal bounds in summarily dismissing appellant's motion for reconsideration and new trial without any injury as to the truth of the facts alleged therein. Appellant based his motion on the claim that a portion of the land in question either is needed for river bank protection or forms part of permanent timberland. If this claim that any portion of the land in question still forms part of the public forests is true, then possession thereof, however long, cannot convert it into private property and such portion would fall within the exclusive jurisdiction of the Bureau of Forestry and beyond the power and jurisdiction of the cadastral court to register under the torrens system. Indubitably, there should be conservation of the natural resources of the Philippines. The prodigality of the spendthrift who squanders his substance for the pleasure of the fleeting moment must be restrained for the less spectacular but surer policy which protects Nature's wealth for future generations. Such is the wise stand of our Government as represented by the Director of Forestry who, with the Forester for the Government of the United States, believes in 'the control of nature's powers by man for his own good. The land which is public forest land, is part of the public domain, and cannot be appropriated. Before private interests have intervened, the government may decide for itself what portions of the public domain shall be set aside and reserved as forest land. Possession of forest lands, however long, cannot ripen into private ownership. Many have written much, and many more have spoken, and quite often, about the pressing need for forest preservation, conservation, protection, development and reforestation. Not without justification. For, forests constitute a vital segment of any country's natural
emm / LTD cases, page 4

FACTS: In a previous cadastral case,judgment was rendered adjudicating Lot No. 6034 to the spouses Perpetuo Silva and Juana Divinagracia, and ordering its registration in their names. The Director of Forestry, through the Provincial fiscal, filed a Petition for Review of Judgment alleging that Lot No. 6084 awarded to the respondentsspouses Perpetuo Silva and Juana Divinagracia forms a major portion of Parcel No. 1 of the Baia Communal Forest situated at Pamplona, Negros, Oriental and under

resources. It is of common knowledge by now that absence of the necessary green cover on our lands produces a number of adverse or ill effects of serious proportions. Without the trees, watersheds dry up; rivers and lakes which they supply are emptied of their contents. The fish disappear. Denuded areas become dust bowls. As waterfalls cease to function, so will hydroelectric plants. The order denying the petition for review of appellant Director of Forestry is reversed and set aside D. TIMBER LANDS 5. Tan v. Director of Forestry (GR L-24548, 27 Oct. 1983, 125 SCRA 302)

acting on the letter, the Secretary of Agriculture and Natural Resources promulgated an order declaring Ordinary Timber License of Tan having been issued by the Director of Forestry without authority is void ab initio. Tan moved for a reconsideration of the order, but the Secretary of Agriculture and Natural Resources denied the motion. Moreover, on April 11, 1964, the Secretary of Agriculture and Natural Resources dismissed the appeals of Jorge Lao Happick and Ravago Commercial Company for reason that the land grant in contention can adversely affect public interest. April 18, 1964, on the basis of the denial of his motion for reconsideration, Tan filed a case before CFI Manila, special civil action, a petition for certiorari, prohibition and mandamus with preliminary prohibitory injunction. A hearing was held on the petition for the issuance of writ of preliminary injunction, wherein evidence was submitted by all the parties including the intervenors (Ravago Commercial Company, Jorge Lao, Happick and AtanacioMallari), and extensive discussion was held both orally and in writing. On January 20, 1965, the court a quo, from the evidence received, resolved not only the question on the issuance of a writ of preliminary injunction but also the motion to dismiss, declared that the petition did not state a sufficient cause of action, and dismissed the same accordingly. Hence an appeal. ISSUE: WON petitioner Tans issuance of license is valid. HELD: Validity of issuance of Petitioners Ordinary Timber License The director of forestry issued petitioners license without authority as cited in the Gen. Memo Order No. 60, revoking the authority to grant ordinary timber licenses by him. Hence, Petitioners license is declared invalid. A timber license is an instrument by which the State regulates the utilization and disposition of forest resources to the end that public welfare is promoted. A timber license is not a contract within the purview of the due process clause; it is only a license or privilege, which can be validly withdrawn whenever dictated by public interest or public welfare as in this case. Also, it is not a property right, nor does it create a vested right. As provided in paragraph 27 of the rules and regulations included in the ordinary timber license states: "The terms and conditions of this license are subject to change at the discretion of the Director of Forestry, and that this license may be made to expire at an earlier date, when public interests so require", timber licenses are subject to the authority of the Director of Forestry. It is no less true that as a subordinate officer, the Director of Forestry is subject to the control of the Department Head or the Secretary of Agriculture and Natural Resources who, therefore, may impose reasonable regulations in the exercise of the powers of the subordinate officer. The power of control of the Department Head over bureaus and offices includes the power to modify, reverse or set aside acts of subordinate officials. Accordingly, respondent-appellee Secretary of Agriculture and Natural Resources has the authority to revoke, on valid grounds, timber licenses issued by the Director of Forestry. There being supporting evidence (particularly in conserving forest resources/watershed as a matter of public interest), the revocation of petitioner-appellant's timber license was a wise exercise of the power of the respondent- appellee (Secretary of Agriculture and Natural Resources) and therefore, valid.
emm / LTD cases, page 5

FACTS: The Bureau of Forestry issued Notice No. 2087, advertising for public bidding a certain tract of public forest land (consisting of 6,420 hectares situated in Olongapo, Zambales sometime in April 1961. Thereafter, questions arose as to the wisdom of having the area declared as a forest reserve or allow the same to be awarded to the most qualified bidder. President Carlos P. Garcia issued a directive to convert public forestland to a forest reserve for watershed purposes. Secretary Cesar M. Fortich of Agriculture and Natural Resources sustained the findings and recommendations of the Director of Forestry who concluded that "it would be beneficial to the public interest if the area is made available for exploitation under certain conditions. Finally, of the ten persons who submitted proposed the area was awarded to herein petitioner-appellant WenceslaoVinzons Tan, on April 15, 1963 by the Bureau of Forestry (p. 17, CFI rec.). Against this award, bidders Ravago Commercial Company and Jorge Lao Happick filed motions for reconsideration, which were denied by the Director of Forestry on December 6, 1963. General Memorandum Order No. 46, series of 1963, was issued by the Secretary of Agriculture and Natural Resources Benjamin M. Gozon who succeeded Secretary Cesar M. Fortich in office on May 30, 1963. It states that- -the Director of Forestry is hereby authorized to grant (a) new ordinary timber licenses where the area covered thereby is not more than 3,000 hectares each; and (be the extension of ordinary timber licenses for areas not exceeding 5,000 hectares each. Thereafter, Jose Y. Feliciano was appointed as Acting secretary of Agriculture and Natural Resources, replacing secretary Benjamin M. Gozon. He promulgate on December 19, 1963 General memorandum Order No. 60, revoking the authority delegated to the Director of Forestry, under General Memorandum order No. 46, to grant ordinary timber licenses, which order took effect on the same day. December 19, 1963, Ordinary Timber License No. 20-'64 (NEW) dated April 22, 1963, in the name of Wenceslao Vinzons Tan, was signed by then Acting Director of Forestry Estanislao R. Bernal without the approval of the Secretary of Agriculture and Natural Resources. On January 6, 1964, the license was released by the Office of the Director of Forestry w/o the signature of the secretary of Agriculture and Natural Resources as required in General MO No. 60. Ravago Commercial Company in its letter to the Secretary prayed for the revocation of Tans licence since it was irregular, anomalous and contrary to existing forestry laws, rules and regulations. On March 9, 1964,

OTHER ISSUE: WON dismissal for failure to state a cause of action is proper. HELD: As to the dismissal for failure to state a cause of action

affects the property, rights, or interests of the State and not merely those of the officer nominally made party defendant. Certainly, The promotion of public welfare and the protection of the inhabitants near the public forest are property, rights and interest of the State.

E. NATIONAL PARKS The order appealed from is affirmed. 6. 1. Petitioner Tan invoked the general rule that in a motion to dismiss based on insufficiency of cause of action, the facts alleged in the complaint are deemed hypothetically admitted for the purpose of the motion. This is unsustainable. The exception the rule would be that the motion does not admit allegations of which the court will take judicial notice are not true, nor does the rule apply to legally impossible facts, nor to facts inadmissible in evidence, nor to facts which appear by record or document included in the pleadings to be unfoundedThe trial court held that, on the basis of the evidence presented in the hearings, the timber license relied upon by Tan was null and void since it was issued by the Director of Forestry without authority. Such license being void, Tan's allegation that his right had been violated was false. This Court ruled that the trial court was correct in considering the evidence already presented and in not confining itself to the allegations in Tan's petition. It must be noted that there was a hearing held in the instant case wherein answers were interposed and evidence introduced. In the course of the hearing, petitioner-appellant had the opportunity to introduce evidence in support of allegations in his petition, which he readily availed of. Consequently, he is estopped from invoking the rule that to determine the sufficiency of a cause of action on a motion to dismiss, only the facts alleged in the complaint must be considered. Moreover, petitioner-appellant cannot invoke the rule that, when the ground for asking dismissal is that the complaint states no cause of action, its sufficiency must be determined only from the allegations in the complaint. "The rules of procedure are not to be applied in a very rigid, technical sense; rules of procedure are used only to help secure substantial justice. If a technical and rigid enforcement of the rules were made, their aim would be defeated. In this case, the interest of the public and the welfare of the inhabitants are of primary importance. The area covered by petitioner-appellant's timber license practically comprises the entire Olongapo watershed. It is of public knowledge that watersheds serves as a defense against soil erosion and guarantees the steady supply of water. As a matter of general policy, the Philippine Constitution expressly mandated the conservation and proper utilization of natural resources, which includes the country's watershed. 2. Petitioner did not exhaust all of administrative remedies. Moreover, from the decision of the Secretary of Agriculture and Natural Resources complained of, petitioners had a plain, speedy and adequate remedy by appealing therefrom to the Chief Executive. 3. Petitioner appellant failed to note that his action is a suit against the State, which, under the doctrine of State immunity from suit, cannot prosper unless the State gives its consent to be sued. The respondentsappellees, in revoking the petitioner-appellant's timber license, were acting within the scope of their authority. The State's immunity may be validly invoked against the action as long as it can be shown that the suit really Spouses Palomo v. CA (GR 95608, 21 Jan. 1997, 334 Phil. 357)

The adverse possession which may be the basis of a grant of title in confirmation of imperfect title cases applies only to alienable lands of the public domain. It is elementary in the law governing natural resources that forest land cannot be owned by private persons. It is not registrable and possession thereof, no matter how lengthy, cannot convert it into private property, unless such lands are reclassified and considered disposable and alienable. There is no question that the lots here forming part of the forest zone were not alienable lands of the public domain. As to the forfeiture of improvements introduced by petitioners, the fact that the government failed to oppose the registration of the lots in question is no justification for petitioners to plead good faith in introducing improvements on the lots. SPS. IGNACIO PALOMO and TRINIDAD PASCUAL, and CARMEN PALOMO VDA. DE BUENAVENTURA v CA, REPUBLIC, FAUSTINO J. PERFECTO, RAFFY SANTILLAN, BOY ARIADO, LORENZO BROCALES, SALVADOR DOE, and other DOES (GR 95608, 21 January 1997) ROMERO, J. This pertains to ownership of 15 parcels of land in Tiwi, Albay which form part of the "Tiwi Hot Spring National Park." On 13 June 1913, then Gov. Gen. William Cameron Forbes issued EO 40 which reserved for provincial park purposes some 440,530 sq m of land situated in Naga, Tiwi, Albay pursuant to Act 648 of the Philippine Commission. The then CFI-Albay, USA, ordered the registration of 15 parcels of land covered by EO 40 in the name of Diego Palomo on 9 & 28 Dec. 1916; and 17 Jan. 1917. Diego Palomo donated these parcels of land consisting of 74,872 square meters which were allegedly covered by OCT Nos. 513, 169, 176 and 173 to his heirs, Ignacio and Carmen Palomo two months before his death in April 1937. Claiming that the aforesaid OCTs were lost during the Japanese occupation, Ignacio P. filed a petition for reconstitution with the CFI- Albay on 30 May 1950. The RD of Albay issued TCT Nos. 3911, 3912, 3913 and 3914 sometime in October 1953. On 10 July 1954 Pres. Magsaysay issued Proc. 47 converting the area embraced by EO 40 into the "Tiwi Hot Spring National Park," under the control, management, protection and administration of the defunct Commission of Parks and Wildlife, now a division of the Bureau of Forest Development. The area was never released as alienable and disposable portion of the public domain and, therefore, is neither susceptible to disposition under the provisions of the Public Land Law (CA 141) nor registrable under the Land Registration Act (Act No. 496).
emm / LTD cases, page 6

The Palomos, however, continued in possession of the property, paid real estate taxes thereon and introduced improvements by planting rice, bananas, pandan and coconuts. On 8 April 1971, petitioners mortgaged said parcels of land (TCT 3911, 3912, 3913 and 3914) to guarantee a loan of Php200K from BPI. In 7 May 1974 petitioners filed Civil Case No. T-143 before the then CFI-Albay for Injunction with damages against private respondents Faustino J. Perfecto, Raffy Santillan, Boy Ariado, Lorenzo Brocales, Salvador Doe and other Does who are all employees of the Bureau of Forest Development who entered the land covered by TCT No. 3913 and/or TCT 3914 and cut down bamboos thereat, totally leveling no less than 4 groves worth not less than Php2K. On 11 Oct. 1974, RP filed Civil Case No. T-176 for annulment and cancellation of Certificates of Title involving the 15 parcels of land registered in the name of the petitioners and subject of Civil Case T-143. Impleaded with the petitioners as defendants were the BPI-Legazpi Branch and the Register of Deeds of Albay. The case against BPI was dismissed because the loan of Php200K with the Bank was already paid and the mortgage in its favor cancelled. A joint trial of Civil Case T-143 and T-176 was conducted upon agreement of the parties and on 31 July 1986, the trial court rendered the following decision: IN CIVIL CASE No. T-143, in favor of the defendants and against the plaintiffs, dismissing the complaint for injunction and damages, as it is hereby DISMISSED. Costs against the plaintiffs. In CIVIL CASE No. T-176, in favor of the plaintiffs and against the defendants: (1) Declaring null and void and no force and effect the Order dated 14 Sept. 1953, as well as the OCT Nos. 153, 169, 173 and 176 and TCT Nos. 3911, T3912, T-3913, and T-3914, all of the RD-Albay and all transactions based on said titles. (2) Forfeiting in favor of the plaintiff Government any and all improvements on the lands in question that are found therein and introduced by the defendants; (3) Declaring Lot Nos. 1 to 12, Plan II-9299 and Lots 1, 21, 3 and 4 of Plan II-9205 as part of Tiwi Hot Spring National Park; (4) RD-Albay is hereby ordered to cancel the alleged OCT Nos. 513, 169, 173 and 176, TCT Nos. T-3911, T-3912, T-3913 and T-3914. Costs against the defendants. The court a quo in ruling for the Republic found no sufficient proof that the Palomos have established property rights over the parcels of land in question before the Treaty of Paris which ended the SpanishAmerican War at the end of the century. The court further stated that assuming that the decrees of CFIAlbay were really issued, the Palomos obtained no right at all over the properties because these were issued only when EO 40 was already in force. At this point, we take note that although the Geodetic Engineer of the Bureau of Lands appointed as one of the Commissioners in the relocation survey of the properties stated in his re-amended report that of the 3,384 square meters covered by Lot 2, Plan II-9205, only 1,976 square meters fall within the reservation area, the RTC ordered TCT 3913 covering the entire Lot 21 (sic) Plan II-9205 cancelled.

The petitioners appealed to the CA which affirmed in toto the findings of the lower Court; hence this petition raising the following issues: 1. The respondent CA committed grave abuse of discretion in affirming in toto the decision of the lower court. 2. The declaration of nullity of the petitioners OCTs and subsequent TCTs over the properties in question is contrary to law and jurisprudence on the matter. 3. The forfeiture of all improvements introduced by the petitioners in the premises in favor of the government is against our existing law and jurisprudence. The issues raised essentially boil down to WON the alleged OCTs issued pursuant to the order of the CFI in 1916-1917 and the subsequent TCTs issued in 1953 pursuant to the petition for reconstitution are valid. Petitioners contend that the Treaty of Paris which ended the Spanish-American War at the end of the 19th century recognized the property rights of Spanish and Filipino citizens and the American government had no inherent power to confiscate properties of private citizens and declare them part of any kind of government reservation. They allege that their predecessors in interest have been in open, adverse and continuous possession of the subject lands for 20-50 years prior to their registration in 1916-1917. Hence, the reservation of the lands for provincial purposes in 1913 by then GovGen Forbes was tantamount to deprivation of private property without due process of law. In support of their claim, the petitioners presented copies of a number of decisions of the CFI-Albay which state that the predecessors in interest of the petitioners' father Diego Palomo, were in continuous, open and adverse possession of the lands from 20 to 50 years at the time of their registration in 1916. The SC is not convinced. The Philippines passed to the Spanish Crown by discovery and conquest in the 16th century. Before the Treaty of Paris in 11 April 1899, our lands, whether agricultural, mineral or forest were under the exclusive patrimony and dominion of the Spanish Crown. Hence, private ownership of land could only be acquired through royal concessions which were documented in various forms, such as (1) Titulo Real or Royal Grant," (2) Concesion Especial or Special Grant, (3) Titulo de Compra or Title by Purchase and (4) Informacion Posesoria or Possessory Information title obtained under the Spanish Mortgage Law or under the Royal Decree of 26 January 1889. Unfortunately, no proof was presented that the petitioners' predecessors in interest derived title from an old Spanish grant. Petitioners placed much reliance upon the declarations in various Expediente GLRO Record Decisions in 1916 and 1917 of the CFI-Albay, USA, presided by Judge Isidro Paredes that their predecessors in interest were in open, adverse and continuous possession of the subject lands for 20-50 years. The aforesaid CFI decisions, however, were not signed by the judge but were merely certified copies of notification to Diego Palomo bearing the signature of the clerk of court.
emm / LTD cases, page 7

Moreover, despite claims by the petitioners that their predecessors in interest were in open, adverse and continuous possession of the lands for 20 to 50 years prior to their registration in 1916-1917, the lands were surveyed only in Dec. 1913, the very same year they were acquired by Diego Palomo. Curiously , in February 1913 or 10 months before the lands were surveyed for Diego Palomo, the government had already surveyed the area in preparation for its reservation for provincial park purposes. If the petitioners' predecessors in interest were indeed in possession of the lands for a number of years prior to their registration in 1916-1917, they would have undoubtedly known about the inclusion of these properties in the reservation in 1913. It certainly is a trifle late at this point to argue that the government had no right to include these properties in the reservation when the question should have been raised 83 years ago. As regards the petitioners' contention that inasmuch as they obtained the titles without government opposition, the government is now estopped from questioning the validity of the certificates of title which were granted. As correctly pointed out by the respondent CA, the principle of estoppel, does not operate against the Govt for the act of its agents. Assuming that the CFIs decrees were really issued, the lands are still not capable of appropriation. The adverse possession which may be the basis of a grant of title in confirmation of imperfect title cases applies only to alienable lands of the public domain. There is no question that the lands in the case at bar were not alienable lands of the public domain. As testified by the District Forester, records in the Bureau of Forestry show that the subject lands were never declared as alienable and disposable and subject to private alienation prior to 1913 up to the present. As part of the reservation for provincial park purposes, they form part of the forest zone. It is elementary in the law governing natural resources that forest land cannot be owned by private persons. It is not registrable and possession thereof, no matter how lengthy, cannot convert it into private property, unless such lands are reclassified and considered disposable and alienable. Neither do the tax receipts which were presented in evidence prove ownership of the parcels of land inasmuch as the weight of authority is that tax declarations are not conclusive proof of ownership in land registration cases. Having disposed of the issue of ownership, we now come to the matter regarding the forfeiture of improvements introduced on the subject lands. It bears emphasis that EO 40 was already in force at the time the lands in question were surveyed for Diego Palomo. Petitioners also apparently knew that the subject lands were covered under the reservation when they filed a petition for reconstitution of the lost original certificates of title inasmuch as the blueprint of Survey Work Order No. 21781 of Plan II-9299 approved by the Chief of the Land Registration Office Enrique Altavas in 1953 as a true and correct copy of the Original Plan No. II-9299 filed in the Bureau of Lands dated 11 Sept. 1948 contains the following note, "in conflict with provincial reservation." In any case, petitioners are presumed to know the law and the failure of the government to oppose the registration of the lands in question is no justification for the petitioners to plead good faith in introducing improvements on the lots. Since 1,976 square meters of the 3,384 square meters covered by TCT 3913 fall within the reservation, TCT

3913 should be annulled only with respect to the aforesaid area. Inasmuch as the bamboo groves leveled in TCT 3913 and subject of Civil Case T-143, were within the perimeter of the national park, no pronouncement as to damages is in order. The CAs decision is hereby AFFIRMED with the modification that TCT 3913 be annulled with respect to the 1,976 square meter area falling within the reservation zone. F. 7. FORESHORE AND RECLAIMED LANDS Chavez v. PEA (GR 133250, 9 July 2002, 384 SCRA 152)

The SC expressed that the constitutional intent, both under the 1973 and 1987 Constitutions, is to transfer ownership of only a limited area of alienable land of the pubic domain to a qualified individual. This constitutional intent is safeguarded by the provision prohibiting corporations from acquiring alienable lands of the public domain, since the vehicle to circumvent the constitutional intent is removed. The SCs decision of 9 July 2002 on the instant case states in its summary: a. The 157.84 has. of reclaimed lands comprising Freedom Islands, now covered by certificates of title in PEAs name, are alienable lands of the public domain. PEA may lease these lands to private corps but may not sell or transfer ownership of these lands to private corps. PEA may only sell these lands to Philippine citizens, subject to the ownership limitations in the 1987 Constitution and existing laws. b. The 592.15 has. of submerged areas of Manila Bay remain inalienable natural resources of the public domain until classified as alienable or disposable lands open to disposition and declared no longer needed f or public service. The govt can make such classification and declaration only after PEA has reclaimed these submerged areas. Only then can these lands qualify as agricultural lands of the public domain, which are the only natural resources the government can alienate. In their present state, the 592.15 hectares of submerged areas are inalienable and outside the commerce of man. c. Since the Amended JVA seeks to transfer to AMARI, a private corporation, ownership of 77.34 hectares of the Freedom Islands, such transfer is void for being contrary to Section 3, Article XII of the 1987 Constitution which prohibits private corporations from acquiring any kind of alienable land of the public domain.

d. Since the Amended JVA also seeks to transfer to AMARI ownership of 290.156 hectares of still submerged areas of Manila Bay, such transfer is void for being contrary to Section 2, Article XII of the 1987 Constitution which prohibits the alienation of natural resources other than agricultural lands of the public domain. PEA may reclaim these submerged areas. Thereafter, the government can classify the reclaimed lands as alienable or disposable, and further declare them no longer needed for public service. Still, the transfer of such reclaimed alienable lands of the public domain to AMARI will be void in view of Sec. 3, Art. XII of the 1987 Constitution which prohibits private corporations from acquiring any kind of alienable land of the public domain.
emm / LTD cases, page 8

8.

Republic v. CA and Republic Real Estate Corp. (299 SCRA 199, 25 Nov. 1998)

there was no violation of the 5-year period ban against alienating or encumbering the land, because the land was merely leased and not alienated. It also found that the mortgage to Nenita Co and Antonio Quilatan covered only the improvement and not the land itself. On appeal, the Court of Appeals affirmed the decision of the trial court. Thus, this present petition by the Republic of the Philippines. ISSUES: 1. Whether or not the respondent court erred in holding that the patent granted and certificate of title issued to respondent Morato cannot be cancelled and annulled since the certificate of title becomes indefeasible after one year from the issuance of the title. 2. Whether or not the respondent court erred in holding that the questioned land is part of a disposable public land and not a foreshore land. RULING: The petition is meritorious. 1. Respondent Morato cannot use the doctrine of indefeasibility of her Torrens Title to bar the State from questioning its transfer or encumbrance. The certificate of title issued to her clearly stipulated that its awards was subject to the conditions provided in Sections 118, 119, 121, 122 and 124 of Commonwealth Act No. 141. Consequently, by express provision of Section 118 of Commonwealth Act 141 and in conformity with the policy of the law, any transfer or alienation of a free patent or homestead within five years from the issuance of the patent is proscribed. Such transfer nullifies said alienation and constitutes a cause for the reversion of the property to the State. Since, she violated Section 118, the reversion of the property to the public domain necessarily follows in pursuant to Section 124. Morato's resort to equity, i.e. that the lease was executed allegedly out of the goodness of her heart without any intention of violating the law, cannot help her. Equity, which has been aptly described as "justice outside legality," is applied only in the absence of, and never against, statutory law or judicial rules of procedure. Positive rules prevail over all abstract arguments based on equity contra legem. Moreover, respondents failed to justify their position that the mortgage should not be considered an encumbrance. The questioned mortgage falls squarely within the term "encumbrance" proscribed by Section 118 of the Public Land Act. Verily, a mortgage constitutes a legal limitation on the estate, and the foreclosure of such mortgage would necessarily result in the auction of the property. Even if only part of the property has been sold or alienated within the prohibited period of five years from the issuance of the patent, such alienation is a sufficient cause for the reversion of the whole estate to the State. As a condition for the grant of a free patent to an applicant, the law requires that the land should not be encumbered, sold or alienated within five years from the issuance of the patent. The sale or the alienation of part of the homestead violates that condition. 2.Petitioner correctly contends that the private respondent Morato cannot owned a foreshore land in accordance with law specifically Article 339(1) of the Civil Code and Article 1, case 3 of the Law of Waters in comparison with Article 420 of the Civil Code. In view of
emm / LTD cases, page 9

Here, Justice Puno summarized succinctly the law on the disposition of foreshore lands as follows: Foreshore lands are lands of public dominion intended for public use. So too are lands reclaimed by the government by dredging, filling, or other means. Act 1654 mandated that the control and disposition of the foreshore and lands under water remained in the national government. Said law allowed only the leasing of reclaimed land. The Public Land Acts of 1919 and 1936 also declared that the foreshore and lands reclaimed by the government were to be disposed of to private parties by lease only and not otherwise. Before leasing, however, the Gov.-Gen., upon recommendation of the A&NR Secretary, had first to determine that the land reclaimed was not necessary for the public service. This requisite must have been met before the land could be disposed of. But even then, the foreshore and lands under water were not to be alienated and sold to private parties. The disposition of the reclaimed land was only be lease. The land remained property of the State. 9. Republic, represented by the DIRECTOR OF LANDS v CA, JOSEFINA L. MORATO, SPOUSES NENITA CO and ANTONIO QUILATAN AND THE REGISTER OF DEEDS OF QUEZON PROVINCE (GR 100709, 14 Nov. 1997, 281 SCRA 639)

FACTS: In December 1972, respondent Morato filed a Free Patent Application No. III-3-8186-B on a parcel of land with an area of 1,265 square meters situated at Pinagtalleran, Calauag, Quezon. On January 16, 1974, the patent was approved and the Register of Deeds of Quezon at Lucena City issued on February 4,1974 Original Certificate of Title No. P-17789. Both the free patent and the title specifically mandate that the land shall not be alienated nor encumbered within five years from the date of the issuance of the patent. Subsequently, the District Land Officer in Lucena City, acting upon reports that respondent Morato had encumbered the land in violation of the condition of the patent, conducted an investigation. Thereafter, it was established that the subject land is a portion of the Calauag Bay, five (5) to six (6) feet deep under water during high tide and two (2) feet deep at low tide, and not suitable to vegetation. Then, on October 24, 1974, a portion of land was mortgaged by respondent Morato to respondents Nenita Co and Antonio Quilatan for P 10, 000. Thereafter, the spouses constructed a house on the land and another portion of the land was leased to Perfecto Advicula on February 2, 1976 at P100.00 a month, where a warehouse was constructed. Thus, petitioner filed an amended complaint against the respondents Morato, spouses Nenita Co and Antonio Quilatan and the Register of Deed of Quezon for the cancellation of title and reversion of a parcel of land to public domain, subject to the free patent in favor of the respondent Morato, on the grounds that the land is a foreshore land and was mortgaged and leased within the five-year prohibitory period. After trial, the lower court, rendered a decision dismissing petitioner's complaint. In finding for private respondents, the lower court ruled that

the fact that base on the finding of the Court of Appeals, the subject land has since long become a foreshore land which is binding and conclusive upon this court. Moreover, when the sea moved towards the estate and the tide invaded it, the invaded property became a foreshore land and passed to the realm of the public domain which happen on the subject land in this case. Thus, the subject land in this case, being a foreshore land, should therefore be returned to the public domain in accordance with the law. WHEREFORE, petition GRANTED. This Court hereby REVERSES and SETS ASIDE the assailed decision of the Respondent Court and ORDERS the CANCELLATION of Free Patent No. (IV-3) 275) issued to Respondent Morato and the subsequent Original CERTIFICATE OF Title No. P-17789. The subject land therefore REVERTS to the State. G. SUBMERGED AREAS

11.

Director of Forestry v. Villareal (GR L-32266, 27 Feb. 1989, 170 SCRA 598)

THE DIRECTOR OF FORESTRY v RUPERTO A. VILLAREAL (GR L-32266, 27 February 1989) CRUZ, J The basic question is the legal classification of mangrove swamps or manglares. If they are part of our public forest lands, they are not alienable under the Constitution. If they are considered public agricultural lands, they may be acquired under private ownership. The private respondent's claim to the land in question must be judged by these criteria. The said land consists of 178,113 sq m of mangrove swamps in Sapian, Capiz. R. Villareal applied for its registration on 25 Jan. 1949, alleging that he and his predecessors-in-interest (PII) had been in possession of the land for more than 40 years. He was opposed by several persons, including the petitioner on behalf of RP. After trial, the application was approved by the CFICapiz. The decision was affirmed by the CA. The Director of Forestry then came to this Court in a petition for review on certiorari claiming that the land in dispute was forestal in nature and not subject to private appropriation. He asks that the registration be reversed. It should be stressed at the outset that both the petitioner and the private respondent agree that the land is mangrove land. The bone of contention between the parties is the legal nature of mangrove swamps or manglares. The petitioner claims, it is forestal and therefore not disposable and the private respondent insists it is alienable as agricultural land. For a proper background of this case, we have to go back to the Philippine Bill of 1902, one of the earlier American organic acts in the country. By this law, lands of the public domain in the Philippines were classified into 3 grand divisions: agricultural, mineral and timber or forest lands. This classification was maintained in the 1935 Commonwealth Constitution, until it was superseded by the 1973 Constitution, which expanded the classification of public lands to include industrial or commercial, residential, resettlement, and grazing lands and even permitted the legislature to provide for other categories. This provision has been reproduced, but with substantial modifications, in the present Constitution. Under the 1935 Constitution, which was the charter in force when this case arose, only agricultural lands were allowed to be alienated. Their disposition was provided for under CA 141. Mineral and timber or forest lands were not subject to private ownership unless they were first reclassified as agricultural lands and so released for alienation. Considering the definition of mangrove swamps or manglares in the early case of Montano v. Insular Government (1909), Mangrove swamps were thus considered agricultural lands and so susceptible of private ownership. Subsequently, the Philippine Legislature categorically declared, despite the above-cited case, that mangrove swamps form part of the public forests of this country. This it did in the Admin Code of 1917, which became effective on 1 Oct. 1917, thus:

H. MILITARY AND NAVAL RESERVATION

I.

OWNERSHIP OF WATERS

J.

WATERSHED

K. MANGROVE SWAMP 10. Montano v. Insular Government (GR 3714, 26 Jan. 1909, 12 Phil. 572)

This early case, mangrove swamps or manglares were defined by the Court as: ... mud flats, alternately washed and exposed by the tide, in which grows various kindred plants which will not live except when watered by the sea, extending their roots deep into the mud and casting their seeds, which also germinate there. These constitute the mangrove flats of the tropics, which exist naturally, but which are also, to some extent cultivated by man for the sake of the combustible wood of the mangrove and like trees as well as for the useful nipa palm propagated thereon. Although these flats are literally tidal lands, yet we are of the opinion that they cannot be so regarded in the sense in which that term is used in the cases cited or in general American jurisprudence. The waters flowing over them are not available for purpose of navigation, and they may be disposed of without impairment of the public interest in what remains. xxx Under this uncertain and somewhat unsatisfactory condition of the law, the custom had grown of converting manglares and nipa lands into fisheries which became a common feature of settlement along the coast and at the same time of the change of sovereignty constituted one of the most productive industries of the Islands, the abrogation of which would destroy vested interests and prove a public disaster.

emm / LTD cases, page 10

Sec. 1820. Words and phrase defined. - For the purpose of this chapter 'public forest' includes, except as otherwise specially indicated, all unreserved public land, including nipa and mangrove swamps, and all forest reserves of whatever character. It is noteworthy, though, that notwithstanding this definition, the Court maintained the doctrine in the Montano case when 2 years later it held in the case of Jocson v. Director of Forestry: ...the words timber land are always translated in the Spanish translation of that Act (Act of Congress) as terrenos forestales. We think there is an error in this translation and that a better translation would be 'terrenos madereros.' Lumber land in English means land with trees growing on it. The mangler plant would never be called a tree in English but a bush, and land which has only bushes, shrubs or aquatic plants growing on it cannot be called 'timber land. xxx xxx xxx The fact that there are a few trees growing in a manglare or nipa swamps does not change the general character of the land from manglare to timber land. More to the point, addressing itself directly to abovequoted Section 1820, the Court declared: 'In the case of Mapa vs. Insular Government (10 Phil. Rep., 175), this Court said that the phrase agricultural lands as used in Act No. 926 means those public lands acquired from Spain which are not timber or mineral lands. Whatever may have been the meaning of the term 'forestry' under the Spanish law, the Act of Congress of 1 July 1902, classifies the public lands in the Philippine Islands as timber, mineral or agricultural lands, and all public lands that are not timber or mineral lands are necessarily agricultural public lands, whether they are used as nipa swamps, manglares, fisheries or ordinary farm lands. The definition of forestry as including manglares found in the Admin Code of 1917 cannot affect rights which vested prior to its enactment. These lands being neither timber nor mineral lands, the trial court should have considered them agricultural lands. If they are so, then the rights of appellants are fully established by Act No. 926. The doctrine was reiterated still later in Garchitorena Vda. de Centenera v. Obias, promulgated on 4 March 1933, more than 15 years after the effectivity of the Admin Code of 1917. Justice Ostrand declared for a unanimous Court: The opposition rests mainly upon the proposition that the land covered by the application there are mangrove lands, but we think this opposition of the Director of Forestry is untenable, inasmuch as it has been definitely decided that mangrove lands are not forest lands in the sense in which this phrase is used in the Act of Congress. No elaboration was made on this conclusion which was merely based on the cases of Montano and Jocson. And in 1977, the above ruling was reaffirmed in Tongson v. Director of Forestry, with Justice Fernando declaring that the mangrove lands in litis were agricultural in nature. The decision even quoted with approval the statement of the trial court that: ... Mangrove swamps where only trees of mangrove species grow, where the trees are small and sparse, fit only for firewood purposes and the

trees growing are not of commercial value as lumber do not convert the land into public land. Such lands are not forest in character. They do not form part of the public domain. Only last year, in Republic v. De Porkan, the Court, citing Krivenko v. Register of Deeds, reiterated the ruling in the Mapa case that "all public lands that are not timber or mineral lands are necessarily agricultural public lands, whether they are used as nipa swamps, manglares, fisheries or ordinary farm lands. But the problem is not all that simple. As it happens, there is also a line of decisions holding the contrary view. In Yngson v. A&NR Secretary, promulgated in 1983, the Court ruled "that the Bureau of Fisheries has no jurisdiction to dispose of swamp lands or mangrove lands forming part of the public domain while such lands are still classified as forest lands. Four months later, in Heirs of Amunategui v. Director of Forestry, the Court was more positive when it held, again through Justice Gutierrez: The Heirs of Jose Amunategui maintain that Lot No. 885 cannot be classified as forest land because it is not thickly forested but is a 'mangrove swamps.' Although conceding that 'mangrove swamp' is included in the classification of forest land in accordance with Section 1820 of the Revised Admin Code, the petitioners argue that no big trees classified in Sec. 1821 of the said Code as first, second and third groups are found on the land in question. Furthermore, they contend that Lot 885, even if it is a mangrove swamp, is still subject to land registration proceedings because the property had been in actual possession of private persons for many years, and therefore, said land was already 'private land' better adapted and more valuable for agricultural than for forest purposes and not required by the public interests to be kept under forest classification. The petition is without merit. A forested area classified as forest land of the public domain does not lose such classification simply because loggers or settlers may have stripped it of its forest cover. Parcels of land classified as forest land may actually be covered with grass or planted to crops by kaingin cultivators or other farmers. 'Forested lands' do not have to be on mountains or in out-of-the-way places. Swampy areas covered by mangrove trees, nipa palms, and other trees growing in brackish or sea water may also be classified as forest land. The classification is descriptive of its legal nature or status and does not have to be descriptive of what the land actually looks like. Unless and until the land classsified as 'forest' is released in an official proclamation to that effect so that it may form part of the disposable agricultural lands of the public domain, the rules on confirmation of imperfect titles do not apply.' The view was maintained in Vallarta v. IAC, where this Court agreed with the Solicitor General's submission that the land in dispute, which he described as "swamp mangrove or forestal land," were not private properties and so not registerable. This case was decided only twelve days after the De Porkan case. Faced with these apparent contradictions, the Court feels there is a need for a categorical pronouncement that should resolve once and for all the question of whether mangrove swamps are agricultural lands or forest lands. The determination of this question is a function initially
emm / LTD cases, page 11

belonging to the legislature, which has the authority to implement the constitutional provision classifying the lands of the public domain. The legislature having made such implementation, the executive officials may then, in the discharge of their own role, administer our public lands pursuant to their constitutional duty "to ensure that the laws be faithfully executed' and in accordance with the policy prescribed. For their part, the courts will step into the picture if the rules laid down by the legislature are challenged or, assuming they are valid, it is claimed that they are not being correctly observed by the executive. Thus do the three departments, coordinating with each other, pursue and achieve the objectives of the Constitution in the conservation and utilization of our natural resources. In CA 141, the Natl Assembly delegated to the President the function of making periodic classifications of public lands, thus: Sec. 6. The President, upon A&NR Secretarys recommendation, shall from time to time classify lands of public domain into: (a) Alienable or disposable, (b) Lumber, and (c) Mineral lands, and may at any time and in a like manner transfer such lands from one class to another, for the purposes of their administration and disposition. Sec. 7. For the purposes of administration and disposition of alienable or disposable lands, the President, upon A&NR Secs recommendation, shall from time to time declare what lands are open to disposition or concession under this Act. With particular regard to alienable public lands, Section 9 of the same law provides: For the purpose of their administration and disposition, the lands of the public domain alienable or open to disposition shall be classified, according to the use or purposes to which such lands are destined, as follows: (a) Agricultural; (b) Residential, commercial, industrial, or for similar productive purposes; (c) Educational, charitable, or other similar purposes; and (d) Reservations for townsites and for public and quasi-public uses. The President, upon A&NR Secretarys recommendation, shall from time to time make the classifications provided for in this section, and may, at any time and in a similar manner, transfer lands from one class to another. As for timber or forest lands, the Revised Administrative Code states as follows: Sec. 1826. Regulation setting apart forest reservesRevocation of same. - Upon there commendation of the Director of Forestry, with the approval of the Department Head, the President may set apart forest reserves from the public lands and he shall by proclamation declare the establishment of such reserves and the boundaries thereof, and thereafter such forest reserves shall not be entered, sold, or otherwise disposed of, but shall remain as such for forest uses, and shall be administered in the same manner as public forest. The President may in like manner by proclamation alter or modify the boundaries of any forest reserve from time to time, or revoke any such proclamation, and upon such revocation such forest reserve shall

be and become part of the public lands as though such proclamation had never been made. Sec. 1827. Assignment of forest land for agricultural purposes. - Lands in public forest, not including forest reserves, upon the certification of the Director of Forestry that said lands are better adapted and more valuable for agricultural than for forest purposes and not required by the public interests to be kept under forest, shall be declared by the Department Head to be agricultural lands. With these principles in mind, we reach the following conclusion: Mangrove swamps or manglares should be understood as comprised within the public forests of the Philippines as defined in the aforecited Section 1820 of the Admin Code of 1917. The legislature having so determined, we have no authority to ignore or modify its decision, and in effect veto it, in the exercise of our own discretion. The statutory definition remains unchanged to date and, no less noteworthy, is accepted and invoked by the executive department. More importantly, the said provision has not been challenged as arbitrary or unrealistic or unconstitutional assuming the requisite conditions, to justify our judicial intervention and scrutiny. The law is thus presumed valid and so must be respected. We repeat our statement in the Amunategui case that the classification of mangrove swamps as forest lands is descriptive of its legal nature or status and does not have to be descriptive of what the land actually looks like. That determination having been made and no cogent argument having been raised to annul it, we have no duty as judges but to apply it. And so we shall. Our previous description of the term in question as pertaining to our agricultural lands should be understood as covering only those lands over which ownership had already vested before the Admin Code of 1917 became effective. Such lands could not be retroactively legislated as forest lands because this would be violative of a duly acquired property right protected by the due process clause. So we ruled again only two months ago in Republic vs. CA, where the possession of the land in dispute commenced as early as 1909, before it was much later classified as timberland. It follows from all this that the land under contention being admittedly a part of the mangrove swamps of Sapian, and for which a minor forest license had in fact been issued by the Bureau of Forestry from 1920 to 1950, it must be considered forest land. It could therefore not be the subject of the adverse possession and consequent ownership claimed by the private respondent in support of his application for registration. To be so, it had first to be released as forest land and reclassified as agricultural land pursuant to the certification the Director of Forestry may issue under Section 1827 of the Revised Admin Code. The private respondent invokes the survey plan of the mangrove swamps approved by the Director of Lands, to prove that the land is registerable. It should be plain, however, that the mere existence of such a plan would not have the effect of converting the mangrove swamps, as forest land, into agricultural land. Such approval is ineffectual because it is clearly in officious. The Director of Lands was not authorized to act in the premises. Under the aforecited law, it is the Director of Forestry who has the authority to determine whether forest land is more valuable for agricultural rather than forestry uses, as a basis for its declaration as agricultural land and release for private ownership.
emm / LTD cases, page 12

Thus we held in the Yngson case: It is elementary in the law governing the disposition of lands of the public domain that until timber or forest lands are released as disposable and alienable neither the Bureau of Lands nor the Bureau of Fisheries has authority to lease, grant, sell or otherwise dispose of these lands for homesteads, sales patents, leases for grazing or other purposes, fishpond leases and other modes of utilization. The Bureau of Fisheries has no jurisdiction to administer and dispose of swamp lands or mangrove lands forming part of the public domain while such lands are still classified as forest land or timber land and not released for fishery or other purposes. The same rule was echoed in the Vallarta case, thus: It is elementary in the law governing natural resources that forest land cannot be owned by private persons. It is not registerable. The adverse possession which can be the basis of a grant of title in confirmation of imperfect title cases cannot commence until after the forest land has been declared alienable and disposable. Possession of forest land, no matter bow long cannot convert it into private property.' We find in fact that even if the land in dispute were agricultural in nature, the proof the private respondent offers of prescriptive possession thereof is remarkably meager and of dubious persuasiveness. The record contains no convincing evidence of the existence of the informacion posesoria allegedly obtained by the original transferor of the property, let alone the fact that the conditions for acquiring title thereunder have been satisfied. Nowhere has it been shown that the informacion posesoria has been inscribed or registered in the registry of property and that the land has been under the actual and adverse possession of the private respondent for 20 years as required by the Spanish Mortgage Law. These matters are not presumed but must be established with definite proof, which is lacking in this case. Significantly, the tax declarations made by the private respondent were practically the only basis used by the appellate court in sustaining his claim of possession over the land in question. Tax declarations are, of course, not sufficient to prove possession and much less vest ownership in favor of the declarant. The SC held that the private respondent Villareal has not established his right to the registration of the subject land in his name. Accordingly, the petition must be granted. It is reiterated for emphasis that, conformably to the legislative definition embodied in Section 1820 of the Revised Administrative Code of 1917, which remains unamended up to now, mangrove swamps or manglares form part of the public forests of the Philippines. As such, they are not alienable under the Constitution and may not be the subject of private ownership until and unless they are first released as forest land and classified as alienable agricultural land. The CAs decision is SET ASIDE and the application for registration of title of private respondent is DISMISSED, with cost against him.

emm / LTD cases, page 13

PURPOSE OF TORRENS 12. 13. 14. 15. Legarda v Saleeby (GR 8936, 2 October 1951, 31 Phil 590) Alba v dela Cruz, 17 Phil 49 (1910) Republic v Umali, 171 SCRA 647 (1989) Pino v CA, Nov. 6, 1997

Director of Lands is not applicable to the present case. In Director of Lands, the "land x x x was already private property at the time it was acquired x x x by Acme." In this case, respondent acquired the land on 8 August 1997 from Porting, who, along with his predecessors-in-interest, has not shown to have been, as of that date, in open, continuous, and adverse possession of the land for 30 years since 12 June 1945. In short, when respondent acquired the land from Porting, the land was not yet private property. For Director of Lands to apply and enable a corporation to file for registration of alienable and disposable land, the corporation must have acquired the land when its transferor had already a vested right to a judicial confirmation of title to the land by virtue of his open, continuous and adverse possession of the land in the concept of an owner for at least 30 years since 12 June 1945. REGISTER OF DEEDS VS. CHINA BANKING CORP FACTS: In a criminal case, Alfonso Pangilinan and one Jose Guillermo Chua were charged with qualified theft. Pangilinan and his wife, Belen Sta. Ana executed a a public instrument a Deed of Transfer after admitting his civil liability in favor of his employer, the CBC, in relation to the offense. He ceded and transferred to the bank, a parcel of land located in Manila City, registered in the name of Belen Sta. Ana, married to Alfonso Pangilinan. The deed was presented for registration to the Register of Deeds of the City of Manila, but because the transferee- the CBC was an alien-owned and as such barred from acquiring lands in the Philippines in accordance with the provisions of Sec. 5, Article XIII (Const.). They submitted the matter to Land Registration Commission in Consulta. LRC handed down resolution declaring that the transfer in favor of alien-bank is in contravention of the Constitution. BANKs ARGUMENT: is based on RA 337 Sec. 25, Par (C) Any commercial bank may purchased, hold, convey real estate for the ff. purposes: (c) Such as shall be conveyed to it in satisfaction of debts previously constructed in the course of its dealings. ISSUE: WON appellant an alien-owned bank can acquire ownership of the residential lot by virtue of the deed of Transfer. HELD: No. RATIO: The court deem it quite clear and free from doubt that the debts referred to in this provision are only those resulting from previous loans and other similar transactions made or entered into by a commercial bank in the ordinary course of its business as such. Obviously, whatever civil liability arising from the criminal offense of qualified theft was admitted in favor of appellant bank by its former employee, Alfonso Pangilinan. It was not a debt arising from a loan or similar transaction between the two parties in the ordinary course of Banking business. That the transfer therefore is in contravention of the constitutional prohibition regarding acquisition of real estate by alien. That the transfer is merely temporary does not affect the issue. Prohibition is absolute.

In Director of Lands v. IAC, the Court allowed the land registration proceeding filed by Acme Plywood Veneer Co., Inc. (Acme) for five parcels of land with an area of 481,390 square meters, or 48.139 hectares, which Acme acquired from members of the Dumagat tribe. The issue in that case was whether the title could be confirmed in favor of Acme when the proceeding was instituted after the effectivity of the 1973 Constitution which prohibited private corporations or associations from holding alienable lands of the public domain except by lease not to exceed 1,000 hectares. The Court ruled that the land was already private land when Acme acquired it from its owners in 1962, and thus Acme acquired a registrable title. Under the 1935 Constitution, private corporations could acquire public agricultural lands not exceeding 1,024 hectares while individuals could acquire not more than 144 hectares. In Director of Lands, the Court further ruled that open, exclusive, and undisputed possession of alienable land for the period prescribed by law created the legal fiction whereby the land, upon completion of the requisite period, ipso jure and without the need of judicial or other sanction ceases to be public land and becomes private property. The Court ruled: Nothing can more clearly demonstrate the logical inevitability of considering possession of public land which is of the character and duration prescribed by statute as the equivalent of an express grant from the State than the dictum of the statute itself that the possessor(s) "x x x shall be conclusively presumed to have performed all the conditions essential to a Government grant and shall be entitled to a certificate of title x x x." No proof being admissible to overcome a conclusive presumption, confirmation proceedings would, in truth be little more than a formality, at the most limited to ascertaining whether the possession claimed is of the required character and length of time; and registration thereunder would not confer title, but simply recognize a title already vested. The proceedings would not originally convert the land from public to private land, but only confirm such a conversion already effected by operation of law from the moment the required period of possession became complete. x x x [A]lienable public land held by a possessor, personally or through his predecessors-in-interest, openly, continuously and exclusively for the prescribed statutory period of (30 years under The Public Land Act, as amended) is converted to private property by the mere lapse or completion of said period, ipso jure. Following that rule and on the basis of the undisputed facts, the land subject of this appeal was already private property at the time it was acquired from the Infiels by Acme. Acme thereby acquired a registrable title, there being at the time no prohibition against said corporation's holding or owning private land. x x x.

emm / LTD cases, page 14

RP, petitioner, vs. HON. REYNALDO M. ALON, Presiding Judge of the Regional Trial Court, Silay City, Branch 40, EDUARDO L. CLAPAROLS, CELIA JALANDONI, CARMITA L. CLAPAROLS, ALBERTO BALCELLS, SAGRARIO MEDINA VDA. DE CLAPAROLS, EULALIA L. CLAPAROLS and FRANCISCO ROSELLO, respondents. G.R. No. 83804 July 18, 1991 FACTS: Civil Case No. 838 was instituted by the Republic in the CFI at Silay City for the purpose of nullifying and canceling resurvey/subdivision-consolidation plans, and certificates of title issued on the basis thereof to the private respondents, on the theory that they embraced increases in or expansions of the original areas obtained by said respondents in violation of law, and said expanded areas form part of the unclassified public forest, not subject to private appropriation. The private respondents were indisputably the registered co-owners of two big tracts of land. Both these lots were resurveyed, and the resurvey plans were approved by the Land Registration Commissioner. The resurvey resulted in the expansion of the original areas of the lots. Thereafter, the owners caused the subdivision of these two (2) expanded lots into smaller lots. The private respondents' claim of title to the expanded areas was founded on the proposition that the same were alluvial in character and therefore accrued to them as riparian owners in accordance with Article 457 of the Civil Code. ISSUE: HELD: The significance and import of the Trial Court's challenged Order of May 12, 1988, as well as of its decision of February 6, 1987, seem to the Court to be sufficiently clear. Both the decision and the order declared that at the very least the evidence of the private respondents established prima facie that they are owners of the expanded areas in question pursuant to Article 475 of the Civil Code and in virtue of open, continuous and exclusive possession of the land for more than 50 years in concept of owners but that the procedure under Section 112 of the Land Registration Act (Act No. 496) by which they succeeded in obtaining title over said areas was incorrect, and that title should issue in their favor only in virtue of regular, original registration proceedings in accordance with the same law in the course of which, as pointed out in Republic v. Heirs of Abrille, supra, "the following requisites should all be satisfied: 1. Survey of land by the Bureau of Lands or a duly licensed private surveyor; 2. Filing of application for registration by the applicant; 3. Setting of the date for the initial hearing of the application by the Court; 4. Transmittal of the application and the date of initial hearing together with all the documents or other evidences attached thereto by the Clerk of Court to the Land Registration Commission; 5. Publication of a notice of the filing of the application and date and place of the hearing in the Official Gazette;

6. Service of notice upon contiguous owners, occupants and those known to have interests in the property by the sheriff; 7. Filing of answer to the application by any person whether named in the notice or not; 8. Hearing of the case by the Court; 9. Promulgation of judgment by the Court; 10. Issuance of the decree by the Court declaring the decision final and instructing the Land Registration Commission to issue a decree Entry of the decree of registration in the Land Registration Commission; 12. Sending of copy of the decree of registration to the corresponding Register of Deeds; and 13. Transcription of the decree of registration book and the issuance of the owner's duplicate original certificate of title to the applicant by the Register of Deeds, upon payment of the prescribed fees. This is indeed what the respondents have done. They have commenced original registration proceedings for the registration of their title over the expanded areas. Their institution of said proceedings pursuant to the decision and order of the Trial Court did not, of course, foreclose the right and option of the Republic to oppose their claim of ownership over those expanded areas, and show the areas to be in truth unclassified forest. But these issues whether or not the land is alluvial or not, or of public or private domain, and whether or not the private respondents are riparian owners thereof within the contemplation of the Civil Code and have occupied the land as owners for fifty years, are issues proper for ventilation in the registration case now pending before the Regional Trial Court at Silay City. As things stand now, the private respondents have in their favor a judicial pronouncement that they have shown, prima facie at least, that the expanded areas are not of public domain and they have acquired rights of ownership over them; in a word, they have acquired rights of ownership over them, in a word, they have overcome the presumption that the land is within an unclassified public forest; on the other hand, there is no categorical declaration in the judgment in Civil Case No. 838 that the expanded areas belong to the State, surely a condition for its entitlement to a writ of possession 22 thereof. In any event, the issues are obviously not triable before this Court, which must therefore reject the Republic's efforts to have said issues ventilated and resolved on the merits in the action at bar. And since, as the record of Civil Case No. 838 of the Regional Trial Court now stands, no clear right to the possession of the expanded areas is shown to exist in favor of the Republic, the Court must also reject the Republic's submittal that the respondent Trial Court, in refusing to grant its motion for writ of possession, had acted beyond its jurisdiction or with grave abuse of discretion. G.R. No. L-5720 [Escueta v. Director of Lands] MARIANO ESCUETA, petitioner-appellee, vs. THE DIRECTOR OF LANDS, objector-appellant.16 Phil. FACTS: On the 17th of August, 1908, Mariano Escueta filed an application in the Court of Land Registration, soliciting
emm / LTD cases, page 15

the registration, in conformity with the Land Registration Act, of a property or parcel of land of which he was alleged to be the absolute owner. The application states that this land contains 798'34,16 square meters, the description and boundaries of which are given in the detailed plan inclosed therewith; that there is no encumbrance of any kind on the said land, nor any person who may consider that he has a right to or a share in the same, according to the applicant's best knowledge and belief, and that the latter acquired the said property, through purchase, from Ildefonso Ramirez y Apostol, according to the deed ratified by the vendee, on May 8, 1908, before the notary public, Fermin Mariano. Among other particulars, the applicant states that he is the present occupant of the land. The application is accompanied by a plan of the land, a deed of sale executed by the said Ildefonso Ramirez in favor of the applicant, and another instrument executed by Telesfora Apostol in favor of the said Ramirez, on August 25, 2905, before the notary Fernando de la Cantera, recorded on folio 73 under No. 1045, fourth inscription; and by another instrument executed by the spouses Francisco Rojo and Susana Modesto, in favor of the aforementioned Telesfora Apostol, on July 7, 1903, before the notary public Calixto Reyes y Cruz, recorded on folio 72, back, of the registry, under No. 1045, third inscription. Court issued an order decreeing the adjudication and registration of the said property in accordance with the petition of Mariano Escueta y Bernabe. On the 23rd of the same month, the petitioner stated in writing to the court that, after the hearing, he had learned that the true boundaries of his land on its southeastern side were the properties of Susan Marquez, Macaria Villareal, Protasio Cabrera, and of the deceased Gregorio Pineda, these two latter instead of Donato Miguel Castro; that the said Protasio Cabrera was then residing at Boac Marinduque and was presented in this city by his sister, Placida Cabrera, domiciled at No. 152 Calle Ilaya, Tondo, and that the deceased Gregorio Pinda was represented by his judicial administratrix, Ignacia Lopez, residing at No. 105 Calle Folgueras, Tondo; that both representative of the said adjacent property owners agreed to the amendment which he petitioned to have made in the description of the said southeastern side were those given in the writing, and that in view of the amendments specified the area of the land should be 806'40,17 square meters instead of 798'34,16 square meters, as appeared on the plan attached and which was annexed to the application. The petitioner therefore asked the court to admit the amendments aforestated. The Attorney-General, by a writing of March 27, 1909, and for the reasons therein expressed, opposed the amendments solicited by the petitioner, except upon a new description of the land, other due notifications and previous publication. ISSUES: Whether or not new notifications and publications are necessary in cases of the realty has been decreed, the applicant requests an amendment of the description of the land and of its plan, with an increase in area and the statement of the name of a new adjacent property owner not mentioned in the previous notifications and publications, or substituting the same by other names. HELD:

From the whole of the provision contained in Act No. 496, it appears that the procedure required in the case of an application for registration is that denominated in law in rem, that is, one against all persons who may allege any right to the property that is the subject of the application for registration, and the decree of the court granting it constitutes a valid and effective title, not only against all the owners of the adjacent properties who appeared at the trial, but also against all the parties who may have an interest in the land. The alteration or amendment of the plan of the land in question and of its description may affect other persons besides the adjacent property owners Protasio Cabrera and the heirs of Gregorio Pineda: wherefore the agreement of these owners or of their representatives is not sufficient in order to comply with the Land Registration Act, for there might other persons who have a right in rem in the properties of the said adjacent owners who would be injured by the diminution of the area of the property encumbered by such a right in rem. This possible injury to them should be avoided by means of new notification and publications concerning the rectification or amendment desired. Moreover, a third party, who did not appear to allege his right in consequence of the previous publication of the description of the land, in accordance with the original plan, in the belief that the property concerned is one in which he had no interest whatever, would afterwards finds that the said land, but a subsequent decree of the court, has a different situation and different boundaries which affect his right, of which differences he was not duly informed, on account of the omission of the indispensable notification and publication ordered by law. This omission might give rise to possibility of claims being filed against the Government for losses and damages on the grounds that the claimant, without negligence on his part and through the lack of notification and publication of the rectification and amendment of the boundaries and situation of the land, was unable to appear at the trial and was prevented from defending his rights and presenting the proper claim. Such a case should be guarded against as should also a decree of registration of a parcel of tract of land which does not conform exactly to the parcel which is the subject of the proceedings or that the description of the same should not coincide with that set forth in the decree ordering the registration. THE DIRECTOR OF LANDS v. CA, TEODORA ABISTADO, et al. July 1997 Teodoro Abistado filed a petition for original registration of his title over 648 square meters of land under Presidential Decree (PD) No. 1529. However, during the pendency of his petition, applicant died. Hence, his heirs Margarita, Marissa, Maribel, Arnold and Mary Ann, all surnamed Abistado represented by their aunt Josefa Abistado, who was appointed their guardian ad litem, were substituted as applicants. The land registration court in its decision dated June 13, 1989 dismissed the petition "for want of jurisdiction." However, it found that the applicants through their predecessors-in-interest had been in open, continuous, exclusive and peaceful possession of the subject land since 1938. The Court noted that applicants failed to comply with the provisions of Section 23 (1) of PD 1529, requiring the Applicants to publish the notice of Initial
emm / LTD cases, page 16

Hearing (Exh. "E") in a newspaper of general circulation in the Philippines. Exhibit "E" was only published in the Official Gazette (Exhibits "F" and "G"). Consequently, the Court is of the well considered view that it has not legally acquired jurisdiction over the instant application for want of compliance with the mandatory provision requiring publication of the notice of initial hearing in a newspaper of general circulation. ISSUE: Is newspaper publication of the notice of initial hearing in an original land registration case mandatory or directory? HELD: Mandatory. Admittedly, P.D. No. 1529, 23 provides that publication in the OG suffices to confer jurisdiction upon the land registration court. However, the question boils down to whether, absent any publication in a NOGC, the land registration court can validly confirm and register the title of private respondents. We hold in the negative. The law used the term shall in prescribing the work to be done by the Commissioner of Land Registration upon the latters receipt of the court order setting the time for initial hearing. The word denotes an imperative and thus indicates the mandatory character of a statute. P.D. No.1529, 23 requires notice of the initial hearing by means of publication, mailing and posting, all of which must be complied with.If the intent of the law were otherwise, said section would not have stressed in detail the requirements of mailing of notices to all persons named in the petition. Indeed, if mailing of notices is essential, then by parity of reasoning, publication in a NOGC islikewise imperative since the law included such requirement in its detailed provision. Further, a land proceedings is in rem, hence must be validated essentially through publication. It may be asked why publication in a NOGC is mandatory when the law already requiresnotice by publication in the OG, mailing and posting.The reason is due process and the realitythat the OG is not as widely read and circulated and is oftentimes delayed in its circulation

a. Rule 52, Sec. 4, Rules of Court: it isdiscretionary upon this Court to grant a withdrawal of appeal after the briefs have been presented. b. Court cannot evade the constitutional question. An important issue is at stake thus, even if the issue can be resolved by the DOJ circular, Court will still decide the case on its merits lest its silence be interpreted as a permanent offense to the Constitution. c. Granting appeal would be tantamount to Krivenko winning his case through DOJ circular. 2. NO. a. Art. XIII Sec. 1 Constitution: all agricultural, timber & mineral lands of the public domain.belong to the State, and their disposition, exploitation, development or utilization shall be limited to the citizens ofthe Philippines or to corporations or associations at least 60% of the capital of which is owned by such citizens b. 3 Classification lands: - public agricultural/agricultural public - timber - mineral c. Public Land Act No. 926: agricultural public lands are those public lands acquired fromSpain which are neither mineral nor timberlands. d. Residential Lands: neither mineral nor timber, they must be classified as agricultural. e. Ibanez de Aldecoa v. Insular Government: Agricultural land not because its actually used for agricultural purposes but because it can be used for that purpose. f. We need to adopt meaning of words in Constitution during the time it was framed.(McKinney v. Barker) g. Commonwealth Act No. 141 Sec. 58-60:permit sale of residential lots to Filipinos. Thus, residential lots are agricultural lands because only agricultural lands are alienable or disposable according to the Constitution. h. Public Agricultural Land (CA No.141) classified lands as (1)agricultural (2)residential (3) commercial (4)industrial (5)other purposes; gives public agricultural land a particular meaning and Constitution gives it a broad/general meaning i. Pubic Land Act No.2874: allowed aliensto own or lease lands (prior to enactment of Consti) j. Opinion of DOJ Sec. Jose Abad Santos: Residential lots are part of public domain, neither mineral nor timber, thus should be classified as agricultural. It means that landis susceptible to cultivation. k. Judicial, legislative & executive brancheshave always maintained that residential lotsare agricultural lands. l. Art. XIII, Sec. 5 Consti: Save in cases of hereditary succession, no private agricultural land will be transferred or assigned except to individuals, corporations, or assoc. qualified to acquire and hold lands of the public domain in the Philippines. closes only means of aliens to acquire land in the country; applies prohibition of alien acquisition of both public & private agricultural lands.
emm / LTD cases, page 17

KRIVENKO V. REGISTER OF DEEDS MANILA Alexander Krivenko, an alien, bought a residential lot from the Magdalena Estate, Inc., in December1941.He was not able to register the land because it was war time then. After the war, he tried to register the land however both theRegister of Deeds Manila and the Court of First Instance refused the registration. ISSUES: 1. WON Court should grant the Motion to Withdraw the Appeal? 2. WON an alien is entitled to acquire a residential land in our country? HELD: 1. NO. Krivenko submitted a motion to withdraw his appeal perhaps because of a circular from the Dept. of Justice instructing all register of deeds to accept for registration all transfers of residential lots to aliens. Court denied the motion.

m. Commonwealth Act No. 141 used to allow aliens to own land by reciprocity, meaning if their country of origin allows Filipinos to own lands, then they can be allowed to own lands here too. However, after the Constitution was promulgated, even acquisition by reciprocity was not allowed. n. RA 133 allows Filipinos to mortgage private real property to aliens as long as they are prohibited from bidding or taking part in sale of property o. Our lands form part of our heritage thus we should preserve them. We need to nationalize them otherwise foreigners might end up owning them which would make a mockery out of our independence. They can lease lands if they wish. OR if they really want to own a land, they can always acquire Filipino citizenship

not only named Tita Yaptinchay LaO the administratrix of her estate with the right to buy the properties of the estate, but also provided that if the said LaO was legally disqualified from buying she was to be her sole heir. Wills- Justina Santos enjoined her heirs to respect the lease contract made, and the conditional option given, in favor of Wong. ISSUE: WoN the lease contract executed by Santos is valid. HELD: This is a misrepresentation of the grossest sort. The documents were known to the defendant-appellant and her counsel even before the death of Justina Santos. Nor is there anything in the documents that is likely to alter the result we have already reached in this case. With respect to the 1957 codicil, it is claimed that Justina Santos could not have intended by the 99-year lease to give Wong the ownership of the land considering that she had earlier devised the property to Tita Yaptinchay LaO. Without passing on the validity of her testamentary disposition since the issue is one pending before the probate court, it suffices to state here that even granting that Justina Santos had devised the land in dispute to LaO, Justina Santos was not thereby barred or precluded from subsequently giving the land to Wong. The execution of the lease contract which, together with the other contracts, amount to a transfer of ownership to Wong, constitutes an implied revocation of her codicil, at least insofar as the disposition of the land is concerned. As for the 1959 wills, it is said that they manifest a desire to abide by the law, as is evident from the statement therein that Wong's right to buy the land be allowed "anytime he or his children should be entitled to buy lands in the Philippines (i.e., upon becoming Filipino citizens)". It seems obvious, however, that this is nothing but a reiteration of the substance of the lease contract and conditional option to buy which in compensation, as our decision demonstrates, amount to a conveyance, the protestation of compliance with the law notwithstanding. In cases like the one at bar, motives are seldom avowed and avowals are not always candid. The problem is not, however, insuperable, especially as in this case the very witnesses for the defendant-appellant testified that Considering her age, ninety (90) years old at the time and her condition, she is a wealthy woman, it is just natural when she said. "This is what I want and this will be done." In particular reference to this contract of lease, when I said "This is not proper, she said 'you just go ahead, you prepare that, I am the owner, and if there is illegality, I am the only one that can question the illegality.'" The ambition of the old woman before her death, according to her revelation to me, was to see to it that these properties be enjoyed, even to own them, by Wong Heng because Doa Justina told me that she did not have any relatives, near or far, and she considered Wong Heng as a son and his children her grandchildren; especially her consolation in life was when she would hear the children reciting prayers in Tagalog. She was very emphatic in the care of the seventeen (17) dogs and of the maids who helped her much, and she told me to see to it
emm / LTD cases, page 18

PHILIPPINE BANKING CORPORATION, representing the estate of JUSTINIA SANTOS Y CANON FAUSTINO, deceased, plaintiff-appellant, vs. LUI SHE, in her own behalf and as administratrix of the intestate estate of Wong Heng, deceased,defendant-appellant. FACTS: This is the second motion that the defendantappellant has filed relative to this Court's decision of September 12, 1967. Accepting the nullity of the other contracts (Plff Exhs. 4-7), the defendant-appellant nevertheless contended that the lease contract (Plff Exh. 3) is so separable from the rest of the contracts that it should be saved from invalidation. In denying the motion, we pointed to the circumstances that on November 15, 1957, the parties entered into the lease contract (in favor of Wong Heng) for 50 years: that ten days after, they amended the contract so as to make it cover the entire property of Justina Santos; less than a month after, they entered into another contract giving Wong Heng the option to buy the leased premises should his pending petition for naturalization be granted; that on November 18, 1958, after failing to secure naturalization and after finding that adoption does not confer the citizenship of the adopting parent on the adopted, the parties entered into two other contracts extending the lease to 99 years and fixing the period of the option to buy at 50 years. which indubitably demonstrate that each of the contracts in question was designed to carry out Justina Santos' expressed wish to give the land to Wong and thereby in effect place its ownership in alien hands, that "as the lease contract was part of a scheme to violate the Constitution it suffers from the same infirmity that renders the other contracts void and can no more be saved from illegality than the rest of the contracts." The present motion is for a new trial and is based on three documents (1 Codicil and 2 wills) executed by Justina Santos which, so it is claimed, constitute newlydiscovered material evidence: Codicil- Justina Santos

that no one could disturb Wong Heng from those properties. That is why we thought of adoption, believing that thru adoption Wong Heng might acquired Filipino citizenship, being the adopted child of a Filipino citizen. The other points raised in the motion for new trial either have already been disposed of in our decision or are so insubstantial to merit any attention. ACCORDINGLY, the motion for new trial is denied. (lease contract not valid)

The court a quo upheld the validity of the usufruct given to Wanda on the ground that the Constitution covers not only succession by operation of law but also testamentary succession. Supreme Court are of the opinion that the Constitutional provision which enables aliens to acquire private lands, does not extend to testamentary succession for otherwise the prohibition will be for naught and meaningless. Any alien would be able to circumvent the prohibition by paying money to a Philippine landowner in exchange for a devise of a piece of land. This opinion notwithstanding, the Supreme Court uphold the usufruct in favor of Wanda because a usufruct, even though a real right, does not vest or give title to the land in the usufructuary, and it is the vesting of title to land in favor of aliens which is prohibited by the Constitution. Usufruct does not transfer title over land.

TESTATE ESTATE OF JOSE EUGENIO RAMIREZ, MARIA LUISA PALACIOS, Administratrix, petitionerappellee, vs. MARCELLE D. VDA. DE RAMIREZ, ET AL., oppositors, JORGE and ROBERTO RAMIREZ, Legatees, oppositors-appellants. GRN L-27952 February 15, 1982 FACTS The case is about the manner of partitioning the testate estate of Jose Eugenio Ramirez among the principal beneficiaries: his widow Marcelle Demoron de Ramirez; his two grandnephews Roberto and Jorge Ramirez; and his companion Wanda de Wrobleski. The facts are: Jose Eugenio Ramirez, a Filipino, died in Spain in December 1964, with only his widow as compulsory heir. His will was admitted to probate by the Court of First Instance. of Manila, Branch. Maria Luisa Palacios was appointed administratrix of the estate. The widow Marcelle is a French who lives in Paris, while the companion Wanda is an Austrian who lives in Spain. In June 1966, the administratrix submitted a project of partition as follows: the property of the deceased is to be divided into two parts. One part shall go to the widow in satisfaction of her legitime; the other part or "free portion" shall go to Jorge and Roberto Ramirez. Furthermore, 1/3 of the free portion is charged, with the widow's usufruct and the remaining 2/3 with a usufruct in favor of Wanda. Jorge and Roberto opposed the project of partition, one of their grounds is that the grant of a usufruct (legal right to use and enjoy the advantages or profits of another persons property) over real property in the Philippines in favor of Wanda de Wrobleski, who is an alien, violates Section 5, Article XIII of the Philippine Constitution ISSUE Whether or not the usufruct given to Wanda is not contrary to Constitutional prohibition against the acquisition of agricultural land by an alien. RULING The appellants claim that-the usufruct over real properties of the estate in favor of Wanda is void because it violates the constitutional prohibition against the acquisition of lands by aliens. The 1935 Constitution which is controlling provides as follows: "SEC. 5. Save in cases of hereditary succession, no private agricultural land shall be transferred or assigned except to individuals, corporations, or associations qualified to acquire or hold lands of the public domain in the Philippines." (Art. XIII.)

Register of Deeds of Rizal vs Ung Siu Si Temple 97 PHIL 58 (1955) FACTS: The Register of Deeds for the province of Rizal refused to accept for a record a deed of donation executed in due form by Jesus Dy, a Filipino citizen, conveying a land in Caloocan, Rizal in favor of an unregistered religious organization Ung Siu Si Temple; operating through all three trustees all of Chinese nationality. The donation was duly accepted by Yu Juan, a Chinese nationality, founder and deaconess of the Temple, acting in representation and in behalf of the latter and its trustees. The refusal of the Register if Deed was elevated in the Court of First Instance of Manila which upheld the action of the Register of Deeds saying: It appearing from the record of the Consulta that UNG SIU SI TEMPLE is a religious organization whose deaconess, founder, trustees and administrator are all Chinese citizens, this Court is of the opinion and so hold that in view of the provisions of the sections 1 and 5 of Article XIII of the Constitution of the Philippines limiting the acquisition of land in the Philippines to its citizens, or to corporations or associations at least sixty per centum of the capital stock of which is owned by such citizens adopted after the enactment of said Act No. 271, and the decision of the Supreme Court in the case of Krivenko vs. the Register of Deeds of Manila, the deed of donation in question should not be admitted for admitted for registration. Not satisfied with the ruling of the Court of First Instance, counsel for the donee Uy Siu Si Temple has appealed to this Court, claiming: (1) that the acquisition of the land in question, for religious purposes, is authorized and permitted by Act No. 271 of the old Philippine Commission, providing as follows: SECTION 1. It shall be lawful for all religious associations, of whatever sort or denomination, whether incorporated in the Philippine Islands or in the name of other country, or not incorporated at all, to hold land in the Philippine Islands upon which to build churches, parsonages, or educational or charitable institutions. SEC. 2. Such religious institutions, if not incorporated, shall hold the land in the name of
emm / LTD cases, page 19

three Trustees for the use of such associations; . . .. (Printed Rec. App. p. 5.) and (2) that the refusal of the Register of Deeds violates the freedom of religion clause of our Constitution [Art. III, Sec. 1(7)]. ISSUES: WON a deed of donation a parcel of land executed in favor of a religious organization whose founder, trustees and administrator are Chinese citizens should be registered or not. RULING: The Court upheld the decision of the CFI of Manila. The Court held that in view of the absolute terms of section 5, Title XIII, of the Constitution, the provisions of Act No. 271 of the old Philippine Commission must be deemed repealed since the Constitution was enacted, in so far as incompatible therewith. In providing that, Save in cases of hereditary succession, no private agricultural land shall be transferred or assigned except to individuals, corporations or associations qualified to acquire or hold lands of the public domain in the Philippines, the Constitution makes no exception in favor of religious associations. Neither is there any such saving found in sections 1 and 2 of Article XIII, restricting the acquisition of public agricultural lands and other natural resources to "corporations or associations at least sixty per centum of the capital of which is owned by such citizens" (of the Philippines). The fact that the appellant religious organization has no capital stock does not suffice to escape the Constitutional inhibition, since it is admitted that its members are of foreign nationality. The purpose of the sixty per centum requirement is obviously to ensure that corporations or associations allowed to acquire agricultural land or to exploit natural resources shall be controlled by Filipinos; and the spirit of the Constitution demands that in the absence of capital stock, the controlling membership should be composed of Filipino citizens. To permit religious associations controlled by nonFilipinos to acquire agricultural lands would be to drive the opening wedge to revive alien religious land holdings in this country. We cannot ignore the historical fact that complaints against land holdings of that kind were among the factors that sparked the revolution of 1896. As to the complaint that the disqualification under article XIII is violative of the freedom of religion guaranteed by Article III of the Constitution, we are by no means convinced (nor has it been shown) that land tenure is indispensable to the free exercise and enjoyment of religious profession or worship; or that one may not worship the Deity according to the dictates of his own conscience unless upon land held in fee simple. The resolution appealed from is affirmed, with costs against appellant.

FACTS: In a criminal case, Alfonso Pangilinan and one Jose Guillermo Chua were charged with qualified theft. Pangilinan and his wife, Belen Sta. Ana executed a a public instrument a Deed of Transfer after admitting his civil liability in favor of his employer, the CBC, in relation to the offense. He ceded and transferred to the bank, a parcel of land located in the City of Manila, registered to the name of Belen Sta. Ana, married to Alfonso Pangilinan. The deed was presented for registration to the Register of Deeds of the City of Manila, but because the transferee- the CBC was an alien-owned and as such barred from acquiring lands in the Philippines in accordance with the provisions of Sec. 5, Article XIII (Const.). They submitted the matter to Land Registration Commission in Consulta. LRC handed down resolution declaring that the transfer in favor of alien-bank is in contravention of the Constitution. BANKS ARGUMENT: is based on RA 337 Sec. 25, Par (C) Any commercial bank may purchased, hold, convey real estate for the ff. purposes: (c) Such as shall be conveyed to it in satisfaction of debts previously constructed in the course of its dealings. ISSUE: WONappellant an alien-owned bank can acquire ownership of the residential lot by virtue of the deed of Transfer. HELD: No.

RATIO: The court deem it quite clear and free from doubt that the debts referred to in this provision are only those resulting from previous loans and other similar transactions made or entered into by a commercial bank in the ordinary course of its business as such. Obviously, whatever civil liability arising from the criminal offense of qualified theft was admitted in favor of appellant bank by its former employee, Alfonso Pangilinan. It was not a debt arising from a loan or similar transaction between the two parties in the ordinary course of Banking business. That the transfer therefore is in contravention of the constitutional prohibition regarding acquisition of real estate by alien. That the transfer is merely temporary does not affect the issue. Prohibition is absolute. Roman Catholic Apostolic vs. LRC 102 PHIL 546 FACTS: Mateo Rodis, a Filipino citizen and a resident of Davao City, executed a deed of sale of a parcel of land located in the same city in favor of petitioner, with Msgr. Clovis Thibault, a Canadian citizen, as actual incumbent (petitioner is a corporation sole). However, respondent deny the registration of such, absence an affidavit declaring that 60% of the member thereof are Filipinos. A motion for reconsideration was filed, but denied. Hence, this petition for mandamus. Petitioner argues that a corporation sole, irrespective of the citizenship of its incumbent, is not prohibited or disqualified to acquire and hold real property. The Corporation Law and the Canon Law provides that a corporation sole is not the owner but merely the administrator of properties acquired by it. Respondents averred that although it might be true that the petitioner
emm / LTD cases, page 20

REGISTER OF CORPORATION

DEEDS VS.

CHINA

BANKING

is not the owner of the land purchased, yet it can exercise all rights of ownership over it. ISSUE: WON a corporation sole can own lands. HELD: A corporation sole is a special form of corporation usually associated with the clergy, designed to facilitate the exercise of the functions of ownership of the church which was regarded as the property owner. It consists of one person only who are incorporated by law in order to give them some legal capacities and advantages particularly that of perpetuity in which their natural persons they could not have. Both the Corporation Law and the Canon Law provides that a corporation sole is not the owner of the properties that it may acquire but merely administrator thereof. Being such, the constitutional provision requiring 60% Filipino ownership is not applicable. The said provision is limited to its terms to ownership alone and does not extend to control, unless the control over the property affected has been devised to circumvent the real purpose of the constitution. In cases of a corporation sole, in determining whether the said constitutional provision is applicable to corporation sole, the nationality of the constituents of the diocese, and not the nationality of the actual incumbent, must be taken into consideration. This is because it has no nationality, ergo, cannot be considered as aliens. In the case, even if the question of nationality be considered, the aforesaid constitutional requirement is fully met and satisfied, considering that the corporation sole in question is composed of an overwhelmingly majority of Filipinos. Petition granted.

emm / LTD cases, page 21

Potrebbero piacerti anche